Advertisement
If you have a new account but are having problems posting or verifying your account, please email us on hello@boards.ie for help. Thanks :)
Hello all! Please ensure that you are posting a new thread or question in the appropriate forum. The Feedback forum is overwhelmed with questions that are having to be moved elsewhere. If you need help to verify your account contact hello@boards.ie

Sum of torque

Options
245678

Comments

  • Registered Users Posts: 321 ✭✭neufneufneuf


    Maybe like that:

    kHVjoE.png

    All red disks will slow down, so like energy is (w1-w2)², energy increase. Except for 4 corners. But N can be great and there are always 4 corners. Before start, give energy for turn red disks around themselves and around green axis. After, friction is ON, this increase energy of the system.

    ONtOTB.png

    With several layers for cancel problems at corners:

    ivyCuB.png


    HOyLya.png

    here all torques are reported for superior layer but each layer I win energy, 4 disks (4 corners) for first layer, 4 disks second layers, etc. Each layer 4 disks have won. There is a difference of energy.


    With one layer only: 4 corners give 4 torques but 4 disks at corners don't decrease their rotationnal velocity around their center of gravity.
    With 2 layers only: the same for 4 next corners, previous torque are reported at this layer but 4 disks at layers one decrease rotationnal velocity =>energy is bigger
    Wit 3 layers: 4 disks at corners of the second layer win energy

    R=Radius of first square layer (a side of inside square = 2R)
    r= radius of red disk
    F=basic force from friction
    t=time

    1/ With first layer only, there are:
    - 20 torques work 20*T*(w1-w2)t = 2*20*Fr*w2t = 40Fr(w1-w2)t
    - Friction 1 = 40Frw2t
    - 8 torques back (R-r)Fw1t, with R=6r for example, this give -48Frw1t+8Frw1t
    Sum = 40Fr(w1-w2)t + 40Frw2t -48Frw1t+8Frw1t = 0 so no energy from first layer if alone


    2/ With second layer only, there are:
    - 28 torques work 28*T*w2t
    - Friction 2
    - 8 torques back (R+2r-r)Fw1t
    Sum is 0 too like first layer

    3/ With first AND second layers only, there are:
    - 20+28+8 torques work 52*T*(w1-w2)t = 2*56Fr(w1-w2)t so that add 16Frw1t-16Frw2t
    - Friction 1+Friction2 + 8Frw2t
    - 8 torques back (R-r)Fw1t + 8 torques back (R+2r-r)Fw1t

    The difference is 16Frw1t-8Frw2t, the sum is not 0

    Edit : other case:
    I think it's possible to cover a sphere wih a lot of small disk (with different radius), Launch the sphere at w1 with all disk at w2. At t=0, friction is ON. Half disks slow down, other accelerate, but the enery is (w1-w2-dw2)² and (w1+w2+dw2)². With dw2 delta rotationnal velocity won of lost. So, the sum of energy is not constant due to the square function.


  • Registered Users Posts: 321 ✭✭neufneufneuf


    It was an error, the sum is 0 but I have another idea:

    The sum :

    1/ With first layer only, there are:
    - 24 torques work 24*T*(w1-w2)t = 2*24*Fr*(w1-w2)t = 48Fr(w1-w2)t
    - Friction 1 = 48Frw2t
    - 8 torques back (R)Fw1t, with R=6r for example, this give -48Frw1t


    2/ With second layer only, there are:
    - 32 torques work
    - Friction 2 = 64Frw2t
    - 8 torques back (R+2r)Fw1t
    Sum is 0 too like first layer

    3/ With first AND second layers only, there are:
    - 24+32+8 torques work 64*T*(w1-w2)t = 2*64Fr(w1-w2)t add 16Fr(w1-w2)t
    - Friction 1+Friction2 + 8Frw2t add 16Frw2t
    - 8 torques back (R+r)Fw1t + 8 torques back (R+4r)Fw1t add 16Frw1t

    Sum = 0


    Maybe if w2 layer 2 noted w2' is lower than w2 of layer 1 the sum is not 0. I will compute it. I suppose frictions are different for have the same force.

    - 24+32+8 torques, Frt( 24(w1-w2)+64(w1-w2')+16(w1-w2)), don't change if w2'=w2 because force is the same and torque is apply on first layer not the second
    - Friction 1+Friction2 + 2*8Frw2t add 8Fr(w2+w2')t here friction change because it depend of w2 and w2'
    - 8 torques back (R+2r)Fw1t + 8 torques back (R+4r)Fw1t , don't change if w2' not equal to w2 because force is the same

    here the difference exist it is 8Fr(w2+w2')t that can be different of 8Fr(2w2)t

    Difficult to build in practice because friction must change in dymamics, corners's disks must have a friction that change for have the same force even velocity is not the same.

    YVUbpR.png


  • Registered Users Posts: 321 ✭✭neufneufneuf


    I compute work from torques:

    1/ +24+32+8 torques on red disks = Frt(48(w1−w2)+64(w1−w′2)+16(w1−w2))
    2/ Friction : +24Fr(w2+w2)t+32Fr(w′2+w′2)t+8Fr(w2+w′2)t
    3/ -8 torques layer1 - 8 torque layer2 =−8(R+2r)Fw1t−8(R+2r)Fw1t=−8(6r+2r)Fw1t−64Frw1t=−128Frw1t

    The sum of works from torque = 128Frtw1−64Frtw2−64Frtw′2 +56Frtw2+72Frtw′2 −128Frw1t

    The sum = −8Frtw2+8Frtw′2=8Frt(w′2−w2) it's different of 0


    v0kQNf.png


  • Registered Users Posts: 321 ✭✭neufneufneuf


    Images describes how disks can be build for have different friction.

    Before t=0, an external system give energy for rotate disks around themselves and around green axis. At t=0, the external system is OFF and friction is ON. The energy must be conserved but not.


    1DMHfP.png


    TfuGL8.png


  • Registered Users Posts: 321 ✭✭neufneufneuf


    Maybe this idea ?

    s2eo7R.png


    Before t=0, turn at w1 and w2 disk. We need to give energy for that. W1>W2. Ar t=0, walls of disk has friction with air only at red point, this give forces F1 and F2 and reduce w2, like w1>w2, the energy increase, no ?


  • Advertisement
  • Registered Users Posts: 321 ✭✭neufneufneuf


    With external green objects to accelerate, each green object win energy and disk win energy too because w2 decrease.

    Energy of disk = 1/2md²w1²+1/2mr²(w1−w2)²

    with :

    d = lenght of black arm
    r = radius of disk
    m = mass of disk

    tDAUPu.png


  • Registered Users Posts: 321 ✭✭neufneufneuf


    Maybe the formula: Energy of disk = 1/2md²w1²+1/2mr²(w1−w2)² is only true for a ring not a disk but the same idea can be use with a ring.


  • Registered Users Posts: 321 ✭✭neufneufneuf


    Red point are always like that even the fact that w1<>w2.

    0QGaVt.png

    I think the formula is good for ring not sure for disk, you know or have a link for these formulas ? I looked for 10 hours on Internet and I don't find.

    I posted here:

    http://physics.stackexchange.com/questions/143377/one-disk-ring-in-double-rotation-and-sum-of-energy

    A guy reply that the formula is wrong but 2 guys on 2 anothers forum say it's ok for a ring, maybe for a disk.

    https://www.physicsforums.com/threads/kinetic-energy-of-a-disk-that-turn-around-2-axis.778605/


  • Registered Users Posts: 321 ✭✭neufneufneuf


    I drawn trajectories in doted lines. It's possible to have trajectories like that:

    IQi0j9.png

    or trajectories like that if w1 and w2 are well choosen (depends of the radius):

    fol97G.png

    The trajectory must be adapt for have a torque, better than red point, choose P1/P2 points :


    aVJUtK.png

    and adjust the mass of object to shock for have the same force in value.

    I think it's possible to have a disk with a part of mass and another part without mass. The goal is to find 2 points where trajectories are parallel and in other direction like

    OQfHqR.png

    OQfHqR.png


  • Registered Users Posts: 321 ✭✭neufneufneuf


    Disk can be like that:

    ioAKqM.png

    Part with mass and part without mass are glue together.

    And the trajectory is:

    BpljAv.png

    Like that the disk receive only a torque, its energy increase. Purple objects receive energy too. Blue axis don't receive a force.

    The part of no mass don't change the equation: 1/2md²w1²+1/2mr²(w1−w2)² is always true.

    Red point can be place anywhere on yellow disk I think, just find the good position:

    wAXh3g.png


  • Advertisement
  • Registered Users Posts: 321 ✭✭neufneufneuf


    Like that ?

    hX86V3.png


  • Registered Users Posts: 321 ✭✭neufneufneuf


    No like that:

    ec8qeS.png

    Vleft=R1w2−(R1−d)w1
    Vright=R2w2−(R2−d)w1 With:
    R1=10
    R2=4
    w1=10
    w2=2
    d=4
    I find:
    Vleft=-40
    Vright=8


  • Registered Users Posts: 321 ✭✭neufneufneuf


    I made a mistake R1 must be < R2, so value can be:

    Vleft=-R1w2+(R1−d)w1
    Vright=-R2w2+(R2−d)w1 With:
    R1=0.5
    R2=15
    w1=10
    w2=8
    d=4
    I find:
    Vleft=-49
    Vright=-20

    Add a translation in the good direction for all the system : +30

    Vleft=-19
    Vright=10

    There is a torque only on disk. Energy increase.

    fyoyi5.png


  • Registered Users Posts: 321 ✭✭neufneufneuf


    If I take a simple example. Trajectories are in doted lines. F1/F2 are created with a shock from external object. F11/F21 create a torque on ring, this increase the energy of disk. F12/F22 create a force to blue axis, this axis is free to move in space, so forces can't give a torque on axis but only move it, this give energy too. For simplify the problem, think with mass of blue axis like very high compared to the grey ring. I think like that the energy is not constant.

    Ou64kv.png


  • Banned (with Prison Access) Posts: 2,083 ✭✭✭tom_tarbucket


    My head hurts.

    I don't understand why you keep posting when no one replies with any valid answers or comments what so ever.

    you are obviously smart but get the hint, no valid reply in over 6 months


  • Registered Users Posts: 321 ✭✭neufneufneuf


    I hope someone can help me... and when I post this help me to understand. There is a problem with the sum of energy with Algodoo, I try to understand where this energy come from. If axis is free to move in space, there is no counter torque only a movement, this mouvement can be very low with big mass and it's I watched with Algodoo. The center of mass must be big enough. But if you want I don't post here, I will stop. Sorry for your head.


  • Registered Users Posts: 321 ✭✭neufneufneuf


    I forgot a 2 forces, so there is a torque on black arm, this decrease energy. But the force (F12+F22) on blue axis move all the system, this must increase a little its energy, no ? What's the difference between fixed blue axis and free ? Fixed blue axis F12+F22 don't works and free F12+F22 works.

    WkHsns.png


  • Registered Users Posts: 321 ✭✭neufneufneuf




  • Registered Users Posts: 1,169 ✭✭✭dlouth15


    I hope someone can help me... and when I post this help me to understand. There is a problem with the sum of energy with Algodoo, I try to understand where this energy come from.
    The problem is that Algodoo is merely a simulator and as such will have limitations and inaccuracies.

    I built a very simple model a while back in Algodoo with only two components which gained energy over time despite there being no external source of energy.

    Now from this there's two possibilities.

    1. The laws of physics are wrong.

    OR

    2. Algodoo is wrong.

    How do we determine which of these is the case? If we want to test the first possibility we must built an actual physical model and see if it gains energy over time. If it doesn't we are forced to conclude that the second possibility - that Algodoo is wrong - is the case.

    There's no point in constructing more and more elaborate models in Algodoo (or any other software) if it is violating physics with very simple models.


  • Registered Users Posts: 321 ✭✭neufneufneuf


    You're right and it's for that I'm looking for a case in physics for understand where Algodoo find energy (maybe Algodoo is wrong but I hope no). I think about a case that I explain here:

    http://physics.stackexchange.com/questions/143377/one-disk-ring-in-double-rotation-and-sum-of-energy

    if a torque can exist on ring, it's won, the system create energy and my calculations shows trajectories can be like I think, but maybe it's an error...I'm waiting for an answer.

    'the devil is in the detail'


  • Advertisement
  • Registered Users Posts: 1,169 ✭✭✭dlouth15


    You're right and it's for that I'm looking for a case in physics for understand where Algodoo find energy (maybe Algodoo is wrong but I hope no). I think about a case that I explain here:

    http://physics.stackexchange.com/questions/143377/one-disk-ring-in-double-rotation-and-sum-of-energy

    if a torque can exist on ring, it's won, the system create energy and my calculations shows trajectories can be like I think, but maybe it's an error...I'm waiting for an answer.
    Build it. Physically construct the thing. You will have your answer.


  • Registered Users Posts: 321 ✭✭neufneufneuf


    but the energy won is very low...before construct something I prefer understand the theory and verify with maths if it's ok or not.

    It's seems it's not possible to have trajectories like I want. But it's possible to add N system in parallel. Each system give two energy E1 in heating and energy from disk E2. Sum for each disk is 2E1+E2. With N system this will give N(2E1+E2). I need to add energy at 2 last systems, need 2E1, so the sum of energy is N(2E1+E2)-2E1.

    CO9xKo.png


  • Registered Users Posts: 3,457 ✭✭✭Morbert


    I'll let this thread run for a little longer but I'm beginning to question its appropriateness for this board.


  • Registered Users Posts: 1,169 ✭✭✭dlouth15


    but the energy won is very low...before construct something I prefer understand the theory and verify with maths if it's ok or not.
    I see someone's answered the question of the total energy of your system in another thread on stack exchange.

    http://physics.stackexchange.com/questions/143715/is-this-expression-for-the-kinetic-energy-of-a-spinning-disk-revolving-about-a-s/144043#144043

    [latex]\displaystyle{E_{total}&=\frac12 m (\omega_1 d)^2 + \frac12 m (\omega_1-\omega_2)^2 r^2 }[/latex]

    To me this looks correct.

    I think neufneufneuf's point on stackexchange is that if you attach some sort of braking system to the apparatus to lower the rotation of the wheel thereby reducing [latex]\omega_2[/latex], the rotation of the wheel relative to the arm, then the overall system appears to have gained energy according to that equation.

    I think what is being left out here is conservation of angular momentum. If you apply a brake to lower [latex]\omega_2[/latex] then the angular momentum will be transferred to the arm assembly and [latex]\omega_1[/latex], the rotation of the arm, will also be decreased.

    When you calculate the effect of this and plug the result back into the equation you find that the energy has actually gone down.


  • Registered Users Posts: 321 ✭✭neufneufneuf


    In this case, with:

    3niK6h.png

    Works of forces

    F is the value of green or magenta force

    wdisks=+N1/2mr²((w1−w2i)²−(w1−w2f)²)

    with w2f<w2i, w2f=w2 at final, w2i=w2 initial

    Wfriction=2(N−1)Fdw3t with w3 the mean of w2

    WF1=2dF−2dF=0
    WF2=2dF−2dF=0

    Wmagentaforce=−2Fdw3t

    Sum=+N1/2mr²((w1−w2i)²−(w1−w2f)²)+2(N−1)Fdw3t−2Fdw3t

    Sum=+N1/2mr²((w1−w2i)²−(w1−w2f)²)+2(N−2)Fdw3t

    Sum of energy:

    Before t=0, the system (N disks) has the energy N(1/2md²w1²+1/4mr²(w1−w2)²)

    At final, the system has the energy:

    N(1/2md²w1²+1/4mr²(w1−w2)²)+N1/2mr²((w1−w2i)²−(w1−w2f)²)+2(N−2)Fdw3t


    bIDGxn.png


  • Registered Users Posts: 1,169 ✭✭✭dlouth15


    In this case,
    I don't see that as being any different from the simpler example. You are still using a braking mechanism (friction among the rings) to slow down the rotation of the rings. This will have an effect of also reducing the rate of rotation of the entire assembly due to conservation of angular momentum and overall the energy of the system will reduce.

    You can counteract this by inputting mechanical energy to rings at the ends of the system but all you have done is turned mechanical energy into heat.

    I think in general it is very unlikely that overunity will be found in mechanical systems such as this. If it were the case, it would have been found by now purely by accident.

    A more promising avenue to explore might be renewable sources of energy such as wind, solar etc. They don't create energy out of nothing but they do make use of freely available natural sources. Another area would be energy conservation, making do with less energy.


  • Registered Users Posts: 321 ✭✭neufneufneuf


    With one disk, if a torque is applied energy is constant. Here with N systems there is energy from friction, with (N-1) coeficient, energy of kinetics disks increase with N coeficient, F1 and F2 don't need energy, there are only 2 magenta forces that need energy and it is not a coeficient with N. Even magenta forces need more energy (I'm not sure about my calculations) I can choose N at 1000.

    I made a mistake in my calculations:

    Works of forces

    F is the value of green or magenta force

    wdisks=+N1/2mr²((w1−w2i)²−(w1−w2f)²)

    with w2f<w2i, w2f=w2 at final, w2i=w2 initial

    Wfriction=2(N−1)Frw3t with w3 the mean of w2 **************************** I noted 'd' but it is 'r'

    WF1=2dF−2dF=0
    WF2=2dF−2dF=0

    Wmagentaforces=−2Frw3t

    Sum=+N1/2mr²((w1−w2i)²−(w1−w2f)²)+2(N−1)Frw3t−2Frw3t

    Sum=+N1/2mr²((w1−w2i)²−(w1−w2f)²)+2(N−2)Frw3t

    Sum of energy:

    Before t=0, the system (N disks) has the energy N(1/2md²w1²+1/4mr²(w1−w2)²)

    At final, the system has the energy:

    N(1/2md²w1²+1/4mr²(w1−w2)²)+N1/2mr²((w1−w2i)²−(w1−w2f)²)+2(N−2)Frw3t


    This change the result but the sum is not constant. Maybe if you can tell me where in my equations there is a mistake ?

    You speak about conservation of angular momentum but it's possible to think only with forces and works.


  • Registered Users Posts: 1,169 ✭✭✭dlouth15


    You speak about conservation of angular momentum but it's possible to think only with forces and works.
    That is true but sometimes things can get too complicated to calculate in those terms. For example, in collisions the forces repelling objects might be very complex to model, but away from the collision itself, we know that the total momentum of the system (not angular in this case) must be constant throughout the process. If we know the trajectories of the particles prior to collision, then conservation of momentum (along with conservation of energy) can be used to calculate the trajectories after collision.

    In this case we're using friction. We don't know the coefficient of friction involved so we can't work out the forces of friction. But we do know that friction will eventually slow the rings down. We also know that conservation of angular momentum means that this will have an effect on the rotation of the arms in the assembly. From this we can calculate the new energy and show that it is less than it would be if the rings were not under the influence of friction.


  • Registered Users Posts: 321 ✭✭neufneufneuf


    You're right in my calculations I suppose friction is constant, this give force F, in reality F will change with w2. For simplify the problem, and with a theoretical study it's possible to imagine the roughness is changing with time for have the same force F. It's not for change datas but only for simplify calculations. I know in physics you have habits to consider energy constant and angular momentum constant but I try to find a leak in physics so I calculated the sum of work. I don't use Algodoo, I explain an exercice and I tried to resolve it. Maybe you can help to find my error ?


  • Advertisement
  • Registered Users Posts: 1,169 ✭✭✭dlouth15


    You're right in my calculations I suppose friction is constant, this give force F, in reality F will change with w2. For simplify the problem, and with a theoretical study it's possible to imagine the roughness is changing with time for have the same force F. It's not for change datas but only for simplify calculations. I know in physics you have habits to consider energy constant and angular momentum constant but I try to find a leak in physics so I calculated the sum of work. I don't use Algodoo, I explain an exercice and I tried to resolve it. Maybe you can help to find my error ?
    Well, I think one mistake is that you have [latex]\omega_1[/latex], the rotation of the arm, constant. We know from ordinary experience that this will change. This is due to Newton's third law: For every action there is an equal and opposite reaction. You haven't shown how the forces cause this change and calculated the change.


This discussion has been closed.
Advertisement